Exams Questions Papers Sec1 5

download Exams Questions Papers Sec1 5

of 104

description

Examinations

Transcript of Exams Questions Papers Sec1 5

SECTION 11.In the following non planar graph number of independent loop equations are

A.8B.12

C.7D.5

Answer & ExplanationAnswer: Option DExplanation:Number of independent loop equations are given by, L = B - N + 1where L : No. of loop equationsB : No. of branches = 12N : No. of nodes = 8L = 12 - 8 + 1 = 5.View Answer Workspace Report Discuss in Forum

2.A transmission line has a characteristic impedance of 50 and a resistance of 0.1 /m, if the line is distortion less, the attenuation constant (in Np/m) is

A.500B.5

C.0.014D.0.002

Answer & ExplanationAnswer: Option DExplanation:Distortion less a = RG and Z0 = RG .View Answer Workspace Report Discuss in Forum

3.A certain 8 bit uniform quantization PCM system can accommodate a signal ranging from - 1 V to + V. The rms value of the signal is V . The signal to quantization noise ratio is:

A.30 dB

B.46.91 dB

C.40 dB

D.49.92 dB

Answer & ExplanationAnswer: Option DExplanation:(SNR)dB = 1.76 + 6.02nn = 8Thus (SNR)dB = 1.76 + 6.02 x 8 = 49.92 dB.View Answer Workspace Report Discuss in Forum

4.A rectangular waveguide, in dominant TE mode, has dimensions 10 cm x 15 cm. The cut off frequency is

A.10 GHz

B.1 GHz

C.15 GHz

D.25 GHz

Answer & ExplanationAnswer: Option BExplanation:For TE10 mode

View Answer Workspace Report Discuss in Forum

5.A radar receiver has a noise figure of 10 dB at 300 K having a bandwidth of 2.5 MHz. The minimum power it can receive is

A.3.45 x 10-15 W

B.1.38 x 10-15 W

C.7.5 x 10-15 W

D.93.15 x 10-15 W

Answer & ExplanationAnswer: Option DExplanation:Pm = kT0f(F - 1) = 93.15 x 10-15 W.

6.In the circuit shown, in switch S is open for a long time and is closed at t = 0. The current i(t) for t 0+ is

A.i(t) = 0.5 - 0.125e-1000t A

B.i(t) = 1.5 - 0.125e-1000t A

C.i(t) = 0.5 - 0.5e-1000t A

D.i(t) = 0.375e-1000t A

Answer & ExplanationAnswer: Option AExplanation:i(f) = 0.5, i(i) = 0.75i(t) = Vr + (ii - ij)e-1/p = 0.5 - 0.125e-1000t .View Answer Workspace Report Discuss in Forum

7.In a semiconductor material. The hole concentration is found to be 2 x 2.5 x 1015 cm-3. If mobility of carriers is 0.13 m2/ v-s. Then find the current density if electric field intensity is 3.62 x 10-19

A.7.6237 x 10-4 A/cm2

B.7.6237 x 10-5 A/cm2

C.7.6237 x 10-3 A/cm2

D.none of these

Answer & ExplanationAnswer: Option AExplanation:Current density J = E Where = conductivity Given -> = 0.13 m2/v-s = 0.13 x 104 cm2/V sec P = 2.25 x 1015/cm3 We have, ni = 1.5 x 1010 Also n.p. = n = /p

= (1.6 x 10-19 x 0.13 x 104 x 2.25 x 1015) x = (0.468) (4.5 x 1015) = 2.106 x 1015 /cm J = E Current density = 2.106 x 1015 x 3.620 x 10-19 = 7.6237 x 10-4 A/m2.View Answer Workspace Report Discuss in Forum

8.Consider the system with and where p and q are arbitrary real numbers. Which of the following statements about the controllability of the system is true?

A.The system is completely state controllable for any non zero values p and q

B.Only p = 0 and q = 0 result in controllability

C.The system is uncontrollable for all values of p and q

D.We cannot conclude about controllability from the given data

Answer & ExplanationAnswer: Option CExplanation:Use the condition of controllability.View Answer Workspace Report Discuss in Forum

9.A MOSFET has a threshold voltage of 1 V and oxide thickness of 500 x 10-8 [r = 3.9; 0 = 8.85 x 10-14 F/cm, q = 1.6 x 10-19 c]. The region under the gate is ion implanted for threshold voltage tailoring. The base and type of impant required to shift threshold voltage to - 1 V are __________ .

A.8.6 x 1011/cm2, p-type

B.8.6 x 1011/cm2, n-type

C.0.86 x 109/cm2, p-type

D.1.02 x 1012/cm2, n-type

Answer & ExplanationAnswer: Option AExplanation:VT(new) = VT(odd) +

= 6.903 x 10-8 fB = - 8.6 x 1011 The threshold voltage is always negative for p-channel and hence implant is of p-type.View Answer Workspace Report Discuss in Forum

10.What is the propagation constant for air filled wave guide with dimensions a = 1.59" and b = 0.795" at 4.95 GHz?

A.0.6698B.0.7698

C.0.503D.0.6598

Answer & ExplanationAnswer: Option AExplanation:

Here, a = 1.59'' = 40.386 mm = 4.04 cm .

11.Radiation resistance of an antenna is 54 and loss resistance is 6 . If antenna has power gain of 10, then directivity is:

A.9

B.11.11

C.data not sufficient

D.10

Answer & ExplanationAnswer: Option BExplanation:Efficiency of antenna Power gain = 10 Thus directivity .View Answer Workspace Report Discuss in Forum

12.If the power spectral density of stationary random process is a sine-squared function of frequency, the shape of its autocorrelation is

A.

B.

C.

D.

Answer & ExplanationAnswer: Option BExplanation:Since autocorrelation function and power spectral density bears a Fourier transform relation, then since required in frequency domain will five rectangular convolutions in time domain thus it is triangular function.View Answer Workspace Report Discuss in Forum

13.The inverse of given Laplace transform is

A.sin t

B.cos t

C.et

D.e2t

Answer & ExplanationAnswer: Option BExplanation:

s = x2 - ex

x(t) = cos t.View Answer Workspace Report Discuss in Forum

14.Consider the amplitude modulated (AM) signal Ac cos ct + 2 cos mt cos ct For demodulating the signal using envelope detector, the minimum value of Ac should be

A.2B.1

C.0.5D.0

Answer & ExplanationAnswer: Option AExplanation:AC cos ct + 2 cos mt cos ctAC cosctfor envelope detection < 1 < 1 Ac should be at least-2.View Answer Workspace Report Discuss in Forum

15.In a 4 bit weighted resistor D/A converter, the resistor value corresponding to LSB is 32 k. The resistor value corresponding to MSB will be

A.32 K

B.16 K

C.8 K

D.4 K

Answer & ExplanationAnswer: Option DExplanation:2n - 1 R = 32 K&Omeg;n = 4 8R = 32R = 4 K.

16.Find 'X' in the circuit below :

f1(A, B, C, D) = (6, 7, 13, 14);f2(A, B, C, D) = (3, 6, 7);f3(A, B, C, D) = (5, 6, 7, 14, 15)

A.p(0, 1, 2, 3, 4, 5, 6, 7, 8, 9, 10, 11, 12, 13, 15)

B.0

C.(14)

D.1

Answer & ExplanationAnswer: Option CExplanation:f1(A, B, C, D) = (6, 7, 13, 14)f2(A, B, C, D) = (3, 6, 7)f1 f2 = (3, 13, 14)f3 x (f1 f2) = (14) = yX = f1 x y 14.View Answer Workspace Report Discuss in Forum

17.Suppose that the modulating signal is m(t) = 2cos (2pfmt) and the carrier signal is xC(t) = AC cos(2pfct), which one of the following is a conventional AM signal without over modulation?

A.x(t) = Acm(t)cos(2pfct)

B.x(t) = Ac [1 + m(t)]cos (2pfct)

C.

D.x(t) = Accos(2pfmt)cos(2pfct) + Acsin(2pfmt)sin(2pfct)

Answer & ExplanationAnswer: Option CExplanation:

(C) is without over modulation.View Answer Workspace Report Discuss in Forum

18.What are the values of Emax and Emin displayed on the oscilloscope, when a 1 kV P-P carries is modulated to 50%?

A.2 kV, 0.5 kV

B.1 kV, 0.5 kV

C.0.75 kV, 0.25 kV

D.0.5 kV, 1.5 kV

Answer & ExplanationAnswer: Option CExplanation:

Emax - Emin = 0.5 x 1 kV = 0.5 kV2Emax = 1.5 kVi, Emax = 0.75 kVEmin = 0.25 kV.View Answer Workspace Report Discuss in Forum

19.Consider the common emitter amplifier shown below with the following circuit parameters: = 100, gm = 0.3861 A/V, r0 = rp = 259 , Rs = 1 k, RB = 93 k, RC = 250 , RL = 1 kW, C1 = and C2 = 4.7mF.

The resistance seen by the source Vs is

A.258

B.1258

C.93 k

D.

Answer & ExplanationAnswer: Option BExplanation:Zs = Rs + (RB || Brs)rc = 2.475 = 1.258 kV

View Answer Workspace Report Discuss in Forum

20.As the temperature increases the mobility of electrons __________ .

A.decreases because the number of carries increases with increased collisions

B.increases because the number of carries decreases with decreased collisions

C.remains same

D.none of the above

Answer & ExplanationAnswer: Option AExplanation:Mobility of electrons m = 2.5 for electrons and 2.7 for holes in Si.

21.Consider the following 1. Maximum energy of electrons liberated photoelectrically is independent of light intensity2. Maximum energy of electrons liberated photoelectrically varies nonlinearly with frequency of incident light.

A.(1) is true and (2) is false

B.(1) and (2) both are true

C.(1) is false and (2) is true

D.(1) and (2) both are false

Answer & ExplanationAnswer: Option AExplanation:Statement (ii) is false because maximum energy of photons varies linearly with frequency of incident light. It can be shown as follows :

F1 > F2 > F3 j = light intensity = constant Photocurrent Vs Anode voltage with frequency and incident light as a parameter. The light intensity is constant.View Answer Workspace Report Discuss in Forum

22.When phase-lag compensation is used in a system, then gain cross over frequency, bandwidth and undamped frequency are respectively

A.Increased, Increased, Increased

B.Increased, Increased, Decreased

C.Increased, Decreased, Decreased

D.Decreased, Decreased, Decreased

Answer & ExplanationAnswer: Option DExplanation:Phase lag is delay between a correcting signal of control system and response to it.View Answer Workspace Report Discuss in Forum

23.The equivalent form of the logical expression ABC + A BC + ABC + ABC + AB C is

A.C + (A B)

B.(A + B + C)(A + B + C)

C.(A + B + C)(A + B + C)(A + B + C)

D.(A + B + C)(A + B + C)(A + B + C)

Answer & ExplanationAnswer: Option DExplanation:This can be solve by using Boolean identity but using k map it will be more easy

(A + B + C) (A + B + C) (A + B + C).View Answer Workspace Report Discuss in Forum

24.The gradient of any scalar field always yields __________ .

A.a solenoidal field

B.a conservative field

C.an irrotational field

D.none of these

Answer & ExplanationAnswer: Option BExplanation:Dot product is conservative. Gradient is nothing but dot product.View Answer Workspace Report Discuss in Forum

25.In a flash type ADC, employing 15 comparators resolution with 10 volts reference; is expected to be :

A.0.625 V

B.0.666 V

C.0.525 V

D.insufficient data

Answer & ExplanationAnswer: Option AExplanation:Since number of comparators = 2n - 1 = 15; Then number of bits are '4'Resolution

26.Consider the signal flow graph shown in figure below. The gain is :

A.

B.

C.

D.

Answer & ExplanationAnswer: Option CExplanation:P1 = abcdL1 = be, L2 = cf, L3 = dg1 = 1

.View Answer Workspace Report Discuss in Forum

27.During transmission over a communicate channel, bit errors occur independently with probability . If a block of 3 bits are transmitted, the probability of at most one bit error is equal to

A.

B.

C.

D.none

Answer & ExplanationAnswer: Option DExplanation:Probability of no-error + probability one error

.View Answer Workspace Report Discuss in Forum

28.If x1[n] x1(z) with ROC = R1 and x2[n] x2(z) with ROC = R2 then ROC for x1[n] + x2[n] will be __________ (where Ri Region of convergence and x1[n] and x2[n] are causal).

A.R1 R2

B.R1 R2

C.(R1 R2) (R1)

D.(R1 R2) (R1)

Answer & ExplanationAnswer: Option AExplanation:R1 R2, Suppose R1 for x1(n) be

Then ROC for x1(n) + x2(n) R1 R2 (R1 < R2)Then ROC for x1(n) + x2(n)R1 R2(R1 < R2)

View Answer Workspace Report Discuss in Forum

29.Consider a unity feedback control system with open-loop transfer function The steady state error of the system due to a unity step input is :

A.zeroB.K

C.1/KD.infinite

Answer & ExplanationAnswer: Option AExplanation:.View Answer Workspace Report Discuss in Forum

30.The analog signal given below is sampled at 1200 samples per second. m(t) = 2 sin 120 pt + 3 sin 240 pt + 4 sin 360 pt + 5 sin 480 pt + 6 sin 600 pt + 7 sin 720 pt. The folding (maximum) frequency is __________ .

A.300 Hz

B.360 Hz

C.600 Hz

D.480 Hz

Answer & ExplanationAnswer: Option CExplanation:Folding frequency (max.) = = 600 Hz.

31.If the system T.F. is The differential equation representing the system is

A.

B.

C.

D.none of these

Answer & ExplanationAnswer: Option CExplanation:

(s3 + 2s2 + 5s + 6)y(s) = (s2 + 4) x (s)s3y(s) + 2s2y(s) + 5sy(s) + 6y(s) = s2 x (s) + 4 x (s) Replacing s by and y(s) by y(t) and x(s) by x(t) we get y(t) + 2y(t) + 5y(t) + 6y(t)= x(t) + 4x(t) + 2 + 5 + 6 = + 4x This is required differential equation.View Answer Workspace Report Discuss in Forum

32.In the circuit below, delay of the EXOR and AND

Logic gates are 20 s and 10 s respectively, then the wave y' and y (with initial condition y = 1) will be :

A.

B.

C.

D.

Answer & ExplanationAnswer: Option BExplanation:Since y is at logic '1' initially, y' must be at logic '1'. Also, one input of the EXOR gate is tied to zero therefore 'f' will be directly transferred to the output. The logic gates will introduce gate delays which will lead to the answer .View Answer Workspace Report Discuss in Forum

33.The antenna current of an AM transmitter is 8 A, when only the carrier is sent, but it increases to 8.93 A when the carrier is modulated by a single sine wave. Determine the antenna current when the percent of modulation changes to 0.8

A.9.19 A

B.9.91 A

C.10.56 A

D.8.61 A

Answer & ExplanationAnswer: Option AExplanation:Ic = 8 A m = 0.8

= 8 x 1.149 = 9.19 A.View Answer Workspace Report Discuss in Forum

34.A noiseless 3 KHz channel can transmit binary at the rate __________ .

A.12000 bps

B.10000 bps

C.6000 bps

D.3000 bps

Answer & ExplanationAnswer: Option CExplanation:Maximum data rate for noiseless channel is given by Nyquist Theorem = 2H log2 V bits/sec V - discrete levels (Here Binary i.e. 2) H - Bandwidth Maximum data rate = 2H log2 V = 2(3 KHz) log2 2 = 2 x 3 x 103 x 1 = 6 x 103 = 6000 bps.View Answer Workspace Report Discuss in Forum

35.Find Vx Vy Vz

A.Vx = -6 Vy = 3 Vz = -3

B.Vx = -6 Vy = -3 Vz = 1

C.Vx = 6 Vy = 3 Vz = 3

D.Vx = 6 Vy = 1 Vz = 3

Answer & ExplanationAnswer: Option DExplanation:Apply Mesh analysis

Apply KVL to mesh 2-2 + Vy + 1 = 0 Vy = 1Apply KVL to mesh 1 -8 + Vx + 2 = 0 Vx = 6 VApply KVL to mesh 3-1 + Vz -2 = 0 Vz = 3 V.

36.If x[n] then the sequence x[n] is

A.non-periodic

B.periodic

C.depends on n

D.none of these

Answer & ExplanationAnswer: Option AExplanation:Consider Number of samples per period = 10pThis is not a rational number. Hence signal is non periodic.View Answer Workspace Report Discuss in Forum

37.The impulse response and the excitation function of a linear time invariant causal system are shown in figure (a) and (b) respectively. The output of the system at t = 2 sec is equal to

A.0

B.2

C.

D.

Answer & ExplanationAnswer: Option DExplanation:

The area of the shaded region .View Answer Workspace Report Discuss in Forum

38.Consider a stable and causal system with impulse response h(t) and system function H(S). Suppose H(S) is rational, contains a pole at S = - 2, and does not have a zero at the origin. The location of all other poles and zero is unknown for each of the following statements. Let us determine whether statement is true or false. 1. f[h(t) e-3t] converges2. 3. h(t) has finite duration4. H(s) = H(- s)Choose correct option.

A.1 - True, 2 - False, 3 - True, 4 - False

B.1 - False, 2 - False, 3 - False, 4 - True

C.1 - False, 2 - False, 3 - False, 4 - False

D.1 - True, 2 - can't say, 3 - True, 4 - can't say

Answer & ExplanationAnswer: Option CExplanation:Statement 1 is false, since f{h(t)e3t} corresponds to the value of the Laplace transform of h(t) at s = 3. If this converges, it implies that s = - 3 is in the ROC. A casual and stable system must always have its ROC to the right of all its poles. However, s = - 3 is not to the right of the pole at s = - 2. Statement 2 is false, because it is equivalent to stating that H(0) = 0. This contradicts the fact that H(s) does not have a zero at the origin. Statement 3 is false. If h(t) is of finite duration, then if its Laplace transform has any points in its ROC, ROC must be the entire s-plane. However, this is not consistent with H(s) having a pole at s = - 2. Statement 4 is false. If it were true, then H(s) has a pole at s = - 2, it must also have a pole at s = 2. This is inconsistent with the fact that all the poles of a causal and stable system must be in the left half of the s-plane.View Answer Workspace Report Discuss in Forum

39.The specific gravity of tungsten is 13.8 and its atomic weight is 184.0 Assume that there are two free elements per atom. Then the Fermi level or characteristic energy for the crystal in eV will be

A.8.95 eV

B.-8.95 eV

C.7.326 eV

D.-7.326 eV

Answer & ExplanationAnswer: Option CExplanation:A quantity of any substance equal to its molecular weight in grams is a mole of that substance contains the same number of molecules as one mole of any other substance. This Avogadro's number and equals 6.02 x 1023 molecules per mole. Thus

= 9.03 x 1028 electrons/m3Since for tungsten the atomic and the molecular weights are the same. Therefore for tungsten EF = 3.64 x 10-19()2/3 = 3.64 x 10- 19(9.03 x 1028)2/3 EF = 7.326 eV.View Answer Workspace Report Discuss in Forum

40.An air-filled rectangular waveguide has dimensions 8cm x 10cm . The phase velocity of guided wave at frequency of 3 GHz for TE10 mode is

A.1.5 x 108 m/s

B.5.5 x 108 m/s

C.3.46 x 108 m/s

D.0.18 x 107 m/s

Answer & ExplanationAnswer: Option CExplanation:

VP = 3.46 x 108 m/s.

41.For a = - 1, find the value of V2

A.

B.0

C.V1

D.none of the above

Answer & ExplanationAnswer: Option BExplanation:V1 = i1R1 + i2R2 i1 + ai1 = i2 V2 = i2R2

Putting a = - 1 = 0 V.View Answer Workspace Report Discuss in Forum

42.A unity-feedback control system has the open loop transfer function . If the input system is a unit ramp, the steady state error will be

A.0B.0.5

C.2D.infinity

Answer & ExplanationAnswer: Option CExplanation:

.View Answer Workspace Report Discuss in Forum

43.For a given voltage, four heating coils will produce maximum heat when connected

A.all in parallel

B.all in series

C.with two parallel pairs in series

D.one pair in parallel with the other two in series

Answer & ExplanationAnswer: Option AExplanation:

.View Answer Workspace Report Discuss in Forum

44.Denominator polynomial of a transfer function of certain network is: s3 + s2 + 2s + 24 Then the network is:

A.stable

B.oscillatory

C.unstable

D.depends on numerator polynomial

Answer & ExplanationAnswer: Option CExplanation:Routh array

There is negative number present in first column. Thus network is unstable.View Answer Workspace Report Discuss in Forum

45.A TV picture is to be transmitted over a channel of 6 MHz bandwidth at a 35 dB S/N ratio. The capacity of the channel is

A.50 Mbps

B.100 Mbps

C.36 Mbps

D.72 Mbps

Answer & ExplanationAnswer: Option DExplanation:

W = B = Bandwidth

C = 6 x 106 x 12 = 72 Mbps.

46.The ratio of the diffusion coefficient in a semiconductor has the units

A.V-1B.em.V-1

C.V.cm-1D.V.s

Answer & ExplanationAnswer: Option AExplanation:.View Answer Workspace Report Discuss in Forum

47.Consider the probability density f(x) = ae-b|x| where x is a random variable whose allowable values range from x = - to x = + . P(1 x 2)

A.

B.

C.

D.(e-b - e-2b)

Answer & ExplanationAnswer: Option CExplanation:P(x) = ae-b|x|dxto get apply aebxdx + ae-bxdx, we get = .View Answer Workspace Report Discuss in Forum

48.The amplitude of a pair of composite sinusoidal signal y(n) = x1(n) + x2(n) with x1(n) = sin (5pn) x2(n) = 3 sin (5pn) is __________ .

A.2B.3

C.4D.1

Answer & ExplanationAnswer: Option AExplanation:Amplitude = .View Answer Workspace Report Discuss in Forum

49.The amplifier circuit shown below uses a silicon transistor. The capacitors Cc and CE can be assumed to be short at signal frequency and the effect of output resistance r0 can be ignored. If CE is disconnected from the circuit, which one of the following statements is TRUE?

A.The input resistance Ri increases and the magnitude of voltage gain Av decreases

B.The input resistance Ri decreases and the magnitude of voltage gain Av decreases

C.Both input resistance Ri and the magnitude of voltage gain Av decrease

D.Both input resistance Ri and the magnitude of voltage gain Av increase

Answer & ExplanationAnswer: Option AExplanation:Given circuit after removing CE will behave as current-series feedback. Overall voltage gain will decrease as feedback signal comes into picture and since it is current-series feedback, input impedance increases.View Answer Workspace Report Discuss in Forum

50.A n-channel JFET has IDSS = 2 mA, Gate to source voltage VGS = - 2 V and trans-conductance is 0.5 mW then pinch-off voltage is __________ .

A.- 2 V

B.2 V

C.- 4 V

D.4 V

Answer & ExplanationAnswer: Option CExplanation:

= - 8VP - 16 + 8VP + 16 = 0 (VP + 4)2 = 0 VP = -4 V.

SECTION 21.If we manage to __________ our natural resources, we would leave a better planet for our children.

A.upholdB.restrain

C.CherishD.conserve

Answer & ExplanationAnswer: Option DExplanation:The clue in this sentence is 'If we manage to __________ our natural resources' and 'better planet'. This implies that the blank should be filled by a word which means 'preserve' or 'keep for long time'. Therefore the word 'conserve' is the right answer.View Answer Workspace Report Discuss in Forum

2.(2)3 + (3)4 = (?)5

A.4

B.11

C.none of these

D.not possible

Answer & ExplanationAnswer: Option CExplanation:(2)3 + (3)4 = 2 + 3 = 5 and (10)5 = 51 + 0 = 5 (2)3 + (3)4 = (10)5 .View Answer Workspace Report Discuss in Forum

3.If g(t) = e- p t2 then G(0) is __________ (where g(t) G(f)

A.

B.unity

C.

D.impulse at f = 0

Answer & ExplanationAnswer: Option BExplanation:The signal g(t) given above is a Gaussian pulse, and it satisfies the relation.

Therefore, its fourier transform is same as the signal itself in frequency domain.i.e. G(f) = e-pf2Hence G(f)|f = 0 = e-p(0) = 1.View Answer Workspace Report Discuss in Forum

4.The open loop transfer function of a unity feedback system is: The range of K for which the system is stable is

A.

B.13 > K > 0

C.

D.-6 < K <

Answer & ExplanationAnswer: Option AExplanation:Apply R-H.View Answer Workspace Report Discuss in Forum

5.A discrete time linear shift-invariant system has an impulse response h[n] with h[0] = 1, h[1] = - 1, h[2] - 2, and zero otherwise. The system is given an input sequence x[n] with x[0] - x[2] - 1, and zero otherwise. The number of nonzero samples in the output sequence y[n], and the value of y[2] are, respectively

A.5, 2

B.6, 2

C.6, 1

D.5, 3

Answer & ExplanationAnswer: Option DExplanation:Use convolution to get the result.

6.For a series resonant circuit at low frequency circuit impedance is __________ and at high frequency circuit impedance is __________ Fill in the blanks respectively

A.capacitive, inductive

B.inductive, capacitive

C.resistive, inductive

D.capacitive, resistive

Answer & ExplanationAnswer: Option AExplanation:

From this curve; we can select option (a) as correct answer.View Answer Workspace Report Discuss in Forum

7.The network is

A.First order system and the pole

B.Second order system of the two poles are

C.Second order system of the poles is

D.none of the above

Answer & ExplanationAnswer: Option BExplanation:System T.F. = Characteristic equation: So system is second order system of the two poles are .View Answer Workspace Report Discuss in Forum

8.A signal with 10 V ranged and 1 KHz band width is being digitized using a sample and hold circuit and a 10 bit quantizer. The minimum sampling rate is :

A.62831.9 V/sec

B.125663.8 V/sec

C.31415.95 V/sec

D.None

Answer & ExplanationAnswer: Option AExplanation:Maximum sampling rate = 2pfmax V.View Answer Workspace Report Discuss in Forum

9.The circuit below represents function X(A, B, C, D) as:

A.(5, 8, 13, 14)

B.(0, 1, 2, 3, 4, 6, 7, 8, 10, 11, 12)

C.(5, 9, 13, 14)

D.(0, 1, 2, 3, 4, 5, 6, 7, 8, 10, 11, 12, 15)

Answer & ExplanationAnswer: Option CExplanation:X = A B C(0. D + 0.D) + A B C(0.D + 0.D) + A B C(0.D + 1.D) + A B C(0.D + 0.D)+ A B C(0.D + 0.D) + A B C(0.D + 1.D) + ABC(1.D + 0.D)= (5, 9, 13, 14).View Answer Workspace Report Discuss in Forum

10.The truth table corresponding to the given input digital gate :

A.

B.

C.

D.

Answer & ExplanationAnswer: Option DExplanation:The circuit behaves like NAND gate.

11.The z-transform of a signal converges if and only if

A.

B.

C.

D.

Answer & ExplanationAnswer: Option AExplanation:

For u(n), a right handled sequence, |z| > , |z| = |3z| > 1; |2z| > 1 < 1; < 1.View Answer Workspace Report Discuss in Forum

12.If the characteristic equation of a closed loop system is s2 + 2s + 2, then the system is :

A.over damped

B.critically damped

C.under damped

D.undamped

Answer & ExplanationAnswer: Option CExplanation:1 + G(s) H(s) = 0s2 + 2s + 2 = 0

< 1, under damped.View Answer Workspace Report Discuss in Forum

13.Given digits 2, 2, 3, 3, 3, 4, 4, 4, 4 how many distinct 4 digit numbers greater than 3000 can be formed?

A.50B.51

C.52D.54

Answer & ExplanationAnswer: Option BExplanation:The given digits are 2, 2, 3, 3, 3, 4, 4, 4, 4 we have to find the numbers that are greater than 300 The first digit can be 3 or 4 but not 2.Now, let us fix the first, second and third digits as 3, 2, 2, then the fourth place can be filled in 3 ways.

The number of ways is 3 similarly, we fix first third and fourth place as 3, 2 and 2 respectively (4) so the second place can be filled in 3 ways again,

The number of ways is 3Now, we fix first, second and fourth, previous cases and we obtain the same result. The number of ways is 3 so, the total number of ways is 9 similarly this can done by fixing the numbers as 3 and 4 (instead of 2) and thereby we obtain the a ways eachThe number of numbers starting with 3 is 27Similarly by taking 4 as the first digit we get 27 numbers The number of numbers that are greater than 3000 is 27 + 27 = 54But, 3222, 4222, is not possible as there are only two 2's, 3333 is not possible as there are only three 3's The total number of numbers that are greater than 3000 is 54 - 3 = 51.View Answer Workspace Report Discuss in Forum

14.Consider circuit with 4 : 16 Demux below: Now:1. f1 = (2, 6, 11, 12)2. f2 = (2, 5, 14)3. = p(2, 3, 4, 5, 9, 13, 14)

A.only 1 and 3 are correct

B.only 1 is correct

C.only 2 is correct

D.2 and 3 are correct

Answer & ExplanationAnswer: Option DExplanation:f1 = y2 + y6 + y11 + y12 and f2 = y1 + yl0 + yl3 f1(m, n, o, p) = (3, 4, 9, 13) f2(m, n, o, p) = (2, 5, 14) = (2, 3, 4, 5, 9, 13, 14).View Answer Workspace Report Discuss in Forum

15.The value of the integral is

A.

B.

C.

D.

Answer & ExplanationAnswer: Option AExplanation:Compare this with where s = 3.

16.The following circuit can be represented as :

A.C

B.f(A, B, C) = (0, 1, 2, 3, 4, 5, 6, 7)

C.(A B)C

D.C

Answer & ExplanationAnswer: Option DExplanation:

View Answer Workspace Report Discuss in Forum

17.A two port network is described by the relation V1 = 2I1 + 3V2 I2 = - I1 + 2V2 Then Z-parameter of such network is

A.

B.

C.

D.

Answer & ExplanationAnswer: Option DExplanation:

View Answer Workspace Report Discuss in Forum

18.Find Y in the circuit below :f1(E, F) = (0, 1, 2, 3)f2(A, B, E, F) = (0, 1, 2, 3, 5, 6, 9)f3(A, B, E, F) = (4, 7, 8, 10, 11, 12, 13, 14, 15)

A.f2

B.f2

C.0

D.(0, 1, 2, 3, 5, 6, 9)

Answer & ExplanationAnswer: Option CExplanation:

View Answer Workspace Report Discuss in Forum

19.Calculate the stability factor and change in Ic from 25C to 100C for, = 50, RB/RE = 250, ICO = 19.9 nA for emitter bias configuration

A.42.553, 0.85 F

B.40.91, 0.58 F

C.38.53, 0.85 F

D.41.10, 0.39 F

Answer & ExplanationAnswer: Option AExplanation:

View Answer Workspace Report Discuss in Forum

20.The star equivalent C1 C2, C3 of the delta network is respectively

A.

B.

C.

D.

Answer & ExplanationAnswer: Option AExplanation:

.

21.In a radar system, the range of R1 is achieved at a frequency f1. Then the range R2 at frequency R2 = 8f1 would be __________ [Neglect effect of 1 on the Beam width.]

A.R2 = R1

B.R2 = 4R1

C.R2 = 2R1

D.R2 =

Answer & ExplanationAnswer: Option CExplanation:The range of the radar is directly proportional to the square root of the frequency. R R1 and R2

.View Answer Workspace Report Discuss in Forum

22.With a line charge along z-axis the field intensity is E = ; which of the following figure shows an exact field distribution?

A.

B.

C.

D.

Answer & ExplanationAnswer: Option DExplanation:

The given figure fails to show the symmetry with respect to .

The figure shows symmetry with respect to f, also the length of arrow is decreasing away from the charge shows that magnitude E is decreasing away from line charge.But problem with this figure is longest lines must be shown in most crowded region.

Here we use lines of fixed segments but different thickness. But this attempt also makes the region crowded near origin.

This figure shows compromise. A symmetrical distribution of lines (at every 45) shows azimuthal symmetry and arrowheads show direction.View Answer Workspace Report Discuss in Forum

23.The logic function f(A, B, C) = m(0, 2, 4, 5, 6) can be represented by : 1. 2. 3.

A.1 only

B.1, 2 only

C.1, 3 only

D.1, 2 and 3

Answer & ExplanationAnswer: Option BExplanation:

Now the Variable Entered Map (VEM) is:

View Answer Workspace Report Discuss in Forum

24.In amplitude modulation, carrier signals A cos t has its amplitude A modulated in proportion with message bearing (low frequency) signal m(t). The magnitude of m(t) is chosen to be __________ .

A.less than 1

B.less than or equal to 1

C.more than 1

D.none of these

Answer & ExplanationAnswer: Option BExplanation:For proper recovery of signal |m(t)| 1.View Answer Workspace Report Discuss in Forum

25.For real values of x, the minimum value of the function f(x) exp (x) + exp (- x) is

A.2B.1

C.0.5D.0

Answer & ExplanationAnswer: Option AExplanation:f(x) = ex + e-xf(x) = ex - exf(x) = 0 ex - e-x = 0f'(x) = ex + e-x +ve for x = 0 Thus minimum.Minimum f(x) = e0 + e0 = 2.

26.The logic realized by the circuit shown in figure below is

A.F = A - C

B.F = A + C

C.F = B C

D.F = B C

Answer & ExplanationAnswer: Option BExplanation:F = A BC + ABC + A B C + ABC= B(AC + AC) + B(AC + AC)B(A C) + B(A C)(A C) (B B)A C.View Answer Workspace Report Discuss in Forum

27.An image uses 512 x 512 picture elements. Each of the picture elements can take any of the 8 distinguishable intensity levels. The maximum entropy in the above image will be

A.2097152 bits

B.648 bits

C.786432 bits

D.144 bits

Answer & ExplanationAnswer: Option CExplanation:Each picture element can be represented by 3 bits. Thus total entropy = 512 x 512 x 3 = 786432.View Answer Workspace Report Discuss in Forum

28.A tachometer has a sensitivity of 4 V/1000 rpm. Express the gain constant of the tachometer in the units volts/(rad/sec).

A.0.0489

B.0.0381

C.2

D.none of these

Answer & ExplanationAnswer: Option BExplanation:4 V/100 rpm = V/rad/sec= 0.0381 V/rad/sec.View Answer Workspace Report Discuss in Forum

29.Consider the signal x(n) = 1 + sinn + 3 cosn + cos . Then Fourier series coefficients are __________ .

A.1, - j, + j, j, - j

B.1, + j, - j, - j , + j

C.1, 3 - j, 3 + j, j, - j

D.1, 3 + j, 3 - j, - j, + j

Answer & ExplanationAnswer: Option AExplanation:The signal x(n) = 1 + sinn 3 cos n + cos n + cos is periodic with period N, and we can expand x[n] directly in terms of complex exponentials to obtain

Collecting terms, we find that

Thus for Fourier series coefficients for this example are a0 = 1, a1 = + = -j, a-1 = - = +j, a2 = j, a-2 = -j.View Answer Workspace Report Discuss in Forum

30.A certain system has transfer function a is a parameter. Consider the standard negative unity feedback configuration as shown below.

Which of the following statements is true?

A.The closed loop system in never stable for any value of a a

B.For some positive values of a, the closed loop system is stable, but not for all positive values

C.For all positive values of a, the closed loop system is stable

D.The closed loop system is stable for all values of a, both positive and negative

Answer & ExplanationAnswer: Option CExplanation:Closed loop system transfer functionUse Routh criteria

Thus for all positive value of 'a' this will be stable.

31.If a signal f(t) has energy E, the energy of the signal f(2t) is equal to __________ .

A.EB.E/2

C.E/4D.2E

Answer & ExplanationAnswer: Option BExplanation:Energy content of a signal x(t), E = |f(t)|2 dtNow, E' = |f(2t)|2 dz for signal f(2t)Putting 2t = z, we getE' = |f(t)|2 dz = .View Answer Workspace Report Discuss in Forum

32.For a npn BJT transistor f is 1.64 x 108 Hz. C = 10-14 F; Cp = 4 x 10-13 F and DC current gain is 90. Find fT and gm (f = cut off frequency, C = capacitance, Cp = parasitic capacitance, gm = transconductance, fT = gain BW product)

A.fT = 1.47 x 1010 Hz ; gm = 38 milli mho

B.fT = 1.64 x 108 Hz ; gm = 30 milli mho

C.fT = 1.47 x 109 Hz ; gm = 38 mho

D.fT = 1.33 x 1012 Hz; gm = 0.37 m-mho

Answer & ExplanationAnswer: Option AExplanation:

fT = 90 x 1.64 x 108 = 1.47 x 1010 Hz

gm = 2pfT(C + Cp) = 2 x p x 1.47 x 1010 = (10-14 + 4 x 10-13) gm= 38 m.View Answer Workspace Report Discuss in Forum

33.The impulse response h(t) of a linear time-invariant continuous time system is described by h(t) = exp (at) u(t) + exp (t) u (- t), where u(t) denotes the unit step function, and a and are constants. This system is stable if

A.a is positive and is positive

B.a is negative and is negative

C.a is positive and is negative

D.a is negative and is positive

Answer & ExplanationAnswer: Option DExplanation:h(t) = e+atu(t) + etu(- t)For h(t) to be stableh(t) dt < It will happen when a is negative and is positive.View Answer Workspace Report Discuss in Forum

34.The probability density function of a random variable x is as shown.

The value of A is:

A.

B.

C.

D.

Answer & ExplanationAnswer: Option CExplanation:= 1 = 1 Solve this to get value of A = 1/5.View Answer Workspace Report Discuss in Forum

35.L =

A.108 rad/sec

B.104 rad/sec

C.102 rad/sec

D.10 rad/sec

Answer & ExplanationAnswer: Option DExplanation:

R1 = (10 + 10) = 20 k

.

36.An 8 level encoding scheme is used in a PCM system of 10 kHz channel BW. The channel capacity is

A.80 kbps

B.60 kbps

C.30 kbps

D.18 kbps

Answer & ExplanationAnswer: Option BExplanation:C = 2B log2 M;M = 8 = 23, log2 M = 3 bits= 2 x 10 x 103 x 3 = 60 kbps.View Answer Workspace Report Discuss in Forum

37.For the power amplifier circuit shown below, the maximum power dissipated by both output transistor is

A.9.66 W

B.30.11 W

C.31.66 W

D.33.66 W

Answer & ExplanationAnswer: Option CExplanation:The maximum power dissipated by both output transistor is maximum

View Answer Workspace Report Discuss in Forum

38.For static electric and magnetic fields in an homogenous source-free medium, which of the following represents the correct form of Maxwell's equations?

A..E = 0 x B = 0

B..E = 0.B = 0

C. x E = 0 x B = 0

D. x E = 0.B = 0

Answer & ExplanationAnswer: Option DExplanation:Curl of electric field is zero. Divergence of magnetic field is zero.View Answer Workspace Report Discuss in Forum

39.A periodic rectangular signal, x(t) has the waveform shown in the given figure. Frequency of the fifth harmonic of its spectrum is __________

A.200 Hz

B.250 Hz

C.400 Hz

D.1250 Hz

Answer & ExplanationAnswer: Option DExplanation:The periodic time = 4 ms = 4 x 10-3 s The fundamental frequency Frequency of the 5th harmonic= 250 x 5 = 1250 Hz.View Answer Workspace Report Discuss in Forum

40.An antisymmetric filter having odd number of coefficient may have a performance as

A.low pass

B.high pass

C.band pass

D.all pass

Answer & ExplanationAnswer: Option DExplanation:Consider an anti-symmetric filter response with order 5.h(n) = [h(2), h(1), h(0), - h(1), - h(2)]We know, H(0) = addition of all component values.Hence H(0) = h(0)Similarly H(p) = addition of all component values with alternate negative sign.= h(2) - h(1) + h(0) + h(1) - h(2) = h(0)Hence we get all pass filter.

41.A material has conductivity of 105 mho/m and permeability of 4p x 10-7 H/m. The skin depth at 9 GHz is

A.1.678 m

B.26 m

C.17 m

D.32.32 m

Answer & ExplanationAnswer: Option CExplanation:

View Answer Workspace Report Discuss in Forum

42.A fair coin is tossed independently four times. The probability of the event "the number of time heads shown up is more than the number of times tails shown up" is

A.

B.

C.

D.

Answer & ExplanationAnswer: Option DExplanation:4 heads and 0 tail 3 head and 1 tail.View Answer Workspace Report Discuss in Forum

43.The steady state error of a stable type 0 unity feedback system for a unit step function is :

A.0

B.

C.

D.

Answer & ExplanationAnswer: Option BExplanation:.View Answer Workspace Report Discuss in Forum

44.Find RTH across the terminals A and B

A.52.28

B.58.28

C.52.82

D.58.82

Answer & ExplanationAnswer: Option DExplanation:There are not independent source in the circuit. The Thevenin and Norton equivalent will have 0 A and 0 V sources.To find RTH, a 1 A source is connected as Writing a nodal equations at n,

0.01Vx + 0.003Vx + 0.0067Vx = 10.017Vx = 1 Vx = 58.82 V

View Answer Workspace Report Discuss in Forum

45.Find Y- parameters

A.

B.

C.

D.None of these

Answer & ExplanationAnswer: Option AExplanation:Apply Nodal Analysis

At node V

...(i)At node V2

...(ii)At node V3

Put V3 in equation (i)

Put V3 in equation (ii)

.

46.For a second-order system with the closed-loop transfer function The settling time for 2-percent band, in seconds, is :

A.1.5B.2.0

C.3.0D.4.0

Answer & ExplanationAnswer: Option BExplanation:.View Answer Workspace Report Discuss in Forum

47.An open loop transfer function is given by has

A.one zero at

B.two zeros at

C.three zeros at

D.four zeros at

Answer & ExplanationAnswer: Option CExplanation:The root locus is given below.

From root locus, we can see that there are 3 zeros at .View Answer Workspace Report Discuss in Forum

48.The current i(t) through a 10 resistor in series with an inductance is given by i(t) = 3 + 4 sin (100t + 45) + 4 sin (300t + 60) Amperes. The RMS value of the current and the power dissipated in the circuit are

A.41 A, 410 W respectively

B.35 A, 350 W respectively

C.5 A, 250 W respectively

D.11 A, 1210 W respectively

Answer & ExplanationAnswer: Option CExplanation:

Power = I2R = 25 X 10 = 250 Watts.View Answer Workspace Report Discuss in Forum

49.The cut off voltage for JFET is 5 V. The pinch off voltage is __________ .

A.(5.0)l/2 V

B.2.5 V

C.V

D.5 V

Answer & ExplanationAnswer: Option DExplanation:For JFETID = IDSS (1 - VGS/Vp)20 = (1 - VGS/Vp)2 VGS = VP Vp = 5 V.View Answer Workspace Report Discuss in Forum

50.Assertion (A): Conductors do not permit propagation of waves more than a short distance into the conductor at microwave frequencies.Reason (R): The relaxation time constant for conductors is much small than the period of centimetric EM wave.

A.Both (A) and (R) are true and (R) is the correct explanation of (A)

B.Both (A) and (R) are true but (R) is not the correct explanation of (A)

C.(A) is true but (R) is false

D.(A) is false but (R) is true

Answer & ExplanationAnswer: Option BExplanation:Depth of penetration = , where = relaxation time. would be small, if is large, which itself depends upon relaxation time.t for conductors is of the order of 10-14 s, for = 3 x 10-2 cm period of centimetric EM waves= Therefore, even though t is much smaller than period of centimetric waves but it is not the correct reason for the assertion given.

SECTION 31.The circuit given below is:

A.Band stop filter

B.Band pass filter

C.Oscillator circuit

D.All pass filter

Answer & ExplanationAnswer: Option BExplanation:The given circuit is Band pass filter .View Answer Workspace Report Discuss in Forum

2.Rank of the matrix:

A.3B.2

C.1D.4

Answer & ExplanationAnswer: Option CExplanation:The given matrix is equivalent to , Thus rank is 1.View Answer Workspace Report Discuss in Forum

3.His rather casual remarks on politics __________ his lack of seriousness about the subject.

A.maskedB.belied

C.cherishD.conserve

Answer & ExplanationAnswer: Option CExplanation:The key words in the statement are 'casual remarks' and 'lack of seriousness'. The blank should be filled with a word meaning 'showed' or 'revealed'. Hence, 'betrayed' is the correct answer.View Answer Workspace Report Discuss in Forum

4.The resolution of a 4 bit counting ADC is 0.5 volts. For an analog input of 6.6 volts, the digital output of the ADC will be

A.1011B.1101

C.1100D.1110

Answer & ExplanationAnswer: Option DExplanation:Resolution = = 0.52n = 14 = (1110)n = 4 bits.View Answer Workspace Report Discuss in Forum

5.For the power amplifier circuit shown below, the maximum power dissipated by both output transistor is

A.9.66 W

B.30.11 W

C.31.66 W

D.33.66 W

Answer & ExplanationAnswer: Option CExplanation:The max. power dissipated by both output transistor is max. P2 = = 31.66 W.

6.The maximum percent overshoot with maximum value of first overshoot as 3.5 V and steady state value as 2.575 V is

A.35.92%B.26.43%

C.57.61%D.15.24%

Answer & ExplanationAnswer: Option AExplanation:From the transient response characteristics, Maximum percentage overshoot (MPO) is given as

MPO decides the relative stability of the system.View Answer Workspace Report Discuss in Forum

7.Find Vo(t) for the following circuit.Data: L = 1 H, C = 1 F, R1 = R2 = 1 and zero initial conditions.

A.0 V

B.5 V

C.3 V

D.2 V

Answer & ExplanationAnswer: Option AExplanation:E(s) = L[e(t)] = L[5 sin 3t]

Let V1(s) = Voltage across Land V2(s) = Voltage across R2

V(s) = V1(s) - V2(s)

View Answer Workspace Report Discuss in Forum

8.An XOR gate with 8 variable is as follows:A B C D E F G HThe number of minterms in the Boolean expression is

A.255B.127

C.128D.256

Answer & ExplanationAnswer: Option CExplanation:.View Answer Workspace Report Discuss in Forum

9.The figure shown below is designed to act as a constant current source across the load resistor. The of transistor is 50. What is the value of load current?

A.1.96 mA

B.2 mA

C.2.04 mA

D.None

Answer & ExplanationAnswer: Option AExplanation:Voltage at base = 12 - 5 = 7 vVoltage at emitter = 7 + 0.7 = 7.7 vCurrent through R1 = 2 mAThus current through load = = 1.96 mA.View Answer Workspace Report Discuss in Forum

10.For a given magnetic material operating in linear mode B = 0.06 T. Assume permeability = 51, then magnetization is:

A.44836 A/m

B.46834 A/m

C.946 A/m

D.936 A/m

Answer & ExplanationAnswer: Option BExplanation:

= mH, m = ( - 1) = 51 - 1 = 50 = 50 x H 46834.

11.The circuit is shown in figure.

The current transfer ratio , is

A.

B.

C.

D.

Answer & ExplanationAnswer: Option BExplanation:.View Answer Workspace Report Discuss in Forum

12.Let f(A, B) = A B; then the simplified form of the function f(f(x y, z)w) is

A.(x y z)w

B.x y z w

C.(x y z) + w

D.none of these

Answer & ExplanationAnswer: Option DExplanation:f(A, B) = A Bf(x y, z) = (x y z) = MBut f(MW) is undefined.View Answer Workspace Report Discuss in Forum

13.A system has 12 poles or 2 zeros. Its high frequency asymptote in its magnitude plot has slope of

A.-200 dB/decade

B.-240 dB/decade

C.-280 dB/decade

D.-320 dB/decade

Answer & ExplanationAnswer: Option AExplanation:High frequency asymptotes = - 20(12) + 2(20) = - 200 dB/DECADE.View Answer Workspace Report Discuss in Forum

14.(1 2 1)x = (1 0 0)4 Then x is equal to

A.2

B.3

C.5

D.does not exist

Answer & ExplanationAnswer: Option BExplanation:x2 = 2x + 1 = 16 x2 + 2x - 15 = 0, (x - 3)(x + 5) = 0, x = 3, - 5 Thus x = 3.View Answer Workspace Report Discuss in Forum

15.For the network shown below, the Thevenin voltage and RTH is equal to __________

A.0 V, 58.82

B.0 V, 96.774

C.0 V, 3 K

D.None of these

Answer & ExplanationAnswer: Option AExplanation:Since there is no independent source in the network; VTH = 0. To find RTH apply 1 A current source across A and B.

Apply KCL at node Vx

as

Solving Vx = 58.82 V .

16.Impedance function for the network shown below is:

A.(3s + 1)

B.

C.

D.2s + 3

Answer & ExplanationAnswer: Option CExplanation:

View Answer Workspace Report Discuss in Forum

17.The function f = A B A D E C D E F G F G C E A D B E E B B B B D can be written as

A.A B D

B.A B E

C.A E

D.A E

Answer & ExplanationAnswer: Option DExplanation:A A B B C D D E E F G 2 1 3 3 2 3 1 4 1 2 2 0 A B B 0 D D 0 E 0 0 f = A B B D D E = A 1 1 E = A 0 E = A E.View Answer Workspace Report Discuss in Forum

18.Which of the following functions would have only odd powers of x in its Taylor series expansion about the point x = 0?

A.sin (x3)

B.sin (x2)

C.cos (x3)

D.cos (x2)

Answer & ExplanationAnswer: Option AExplanation:

Thus sin(x3) will have odd powersSin (x2) will have even powersCos (x3) will have even powersCos (x2) will have even powers.View Answer Workspace Report Discuss in Forum

19.Find the maximum power that can be delivered to a load

A.0.025 W

B.2.5 W

C.2.5 W

D.0.25 W

Answer & ExplanationAnswer: Option DExplanation:PL = I2R = x R (i)where zS = 100 + j50zL = 100 - j50Putting the values in eq. (i)PL =X l00 = 0.25 watts.View Answer Workspace Report Discuss in Forum

20.For a integrator circuit, Ri = 100 K, CF = 10 F. The input is a step dc voltage as shown below. Its output will be

A.

B.

C.

D.

Answer & ExplanationAnswer: Option BExplanation:

21.The distance between target and radar is reduced to half. Power received increases by

A.1/16

B.1/4

C.4 times

D.16 times

Answer & ExplanationAnswer: Option DExplanation:.View Answer Workspace Report Discuss in Forum

22.The Causal system given below is __________ H(z) = 6 + z-1 - z-2

A.maximum phase

B.minimum phase

C.mixed phase

D.none of these

Answer & ExplanationAnswer: Option BExplanation:Minimum phaseH(z) = 6 + z-1 - z-1

6z2 + z - 1 = 0

We have zeros at z = 1/3 and z = - 1/2

All the zeroes lies inside the unit circle, it is a min. phase system.View Answer Workspace Report Discuss in Forum

23.Given potential sin cos , find flux density D at (2, 2p, 0)

A.22.12 pC/m2

B.2.212 pC/m2

C.22.12 nC/m2

D.2.212 nC/m2

Answer & ExplanationAnswer: Option AExplanation:

Now E = -VE = V

At r = 2, = p/2; = 0 we have

D = 22.12 PC/m2.View Answer Workspace Report Discuss in Forum

24.The following circuit is implementation of 1. Sum of full adder2. Carry of half adder3. Difference of full subtractor

A.(i) only

B.(iii) only

C.(ii) only

D.(i) and (ii)

Answer & ExplanationAnswer: Option DExplanation:

Now it is evident from the truth tables that sum and differennce are same.View Answer Workspace Report Discuss in Forum

25.The Thevenin impedance Zth between the nodes P and Q in the following circuit is

A.1

B.

C.

D.

Answer & ExplanationAnswer: Option AExplanation:For finding Thevenin equivalentShort voltage sourceOpen circuit sourceNow Thevenin equivalent.

26.If the memory chip size is 256 x 1 bits, then the number of chips required to make up 1 kB (1024) bytes of memory is

A.32B.24

C.12D.8

Answer & ExplanationAnswer: Option AExplanation:.View Answer Workspace Report Discuss in Forum

27.A coil has a designed for high Q performance at a rated voltage and a specified frequency. If the frequency of operation is doubled, and the coil is operated at the same rated voltage, then the Q factor and the active power P consumed by the coil will be affected as follows.

A.P is doubled, Q is halved

B.P is halved, Q is doubled

C.P remains constant, Q is doubled

D.P decreases 4 times, Q is doubled

Answer & ExplanationAnswer: Option DExplanation:2L = 21L, R remains constant Q2 = = 2Q1 i.e. Q is doubled

for a high a coil L >> R

i.e. P decreases 4 times.View Answer Workspace Report Discuss in Forum

28.The number of variables in a function is odd and the number of minterms with even number of 0's are same as the number of minterms with odd number at 1 's, the expression formed can be represented in __________ as well as __________ form.

A.exclusive - OR, NOR

B.equivalence, exclusive - OR

C.AND, coincidence

D.OR, exclusive - OR

Answer & ExplanationAnswer: Option BExplanation:With odd number of variables (n) terms with even number of 0's and terms with odd number of 1 's are the same. Thus the k-map for the above condition with minterms represented by terms with even number of 0's is for n = 3 The expression can be represented by equivalence (EX NOR) or (EX OR) expressions.View Answer Workspace Report Discuss in Forum

29.Find the number of loop equation and number of possible trees for the given graph.

A.7, 2097152

B.7, 262144

C.7.5764801

D.6, 823543

Answer & ExplanationAnswer: Option BExplanation:In any graph, Number of loop equations L = B - N + 1 where B = Number of branches, N = Number of nodes.Given B = 14, N = 8 L = B - N + 1 = 7 The total number of trees possible T = NN - 2 = 88 - 2 = 262144.View Answer Workspace Report Discuss in Forum

30.The number of open right half plane poles of is

A.0B.1

C.2D.3

Answer & ExplanationAnswer: Option CExplanation:Use Routh criteria.

31.The common emitter model is shown below:

The h-parameter of this model are:

A.

B.

C.

D.

Answer & ExplanationAnswer: Option AExplanation:h parameter V1 = (3 + 5)I1 + V2 .View Answer Workspace Report Discuss in Forum

32.In the following limiter circuit, an input voltage V1 = 10 sin 100pt applied. Assume that the diode drop is 0.7 V when it is forward biased. The zener breakdown voltage is 6.8 V

The maximum and minimum values of the output voltage respectively are

A.6.1 V, -0.7 V

B.0.7 V, -7.5 V

C.7.5 V, -0.7 V

D.7.5 V, -7.5 V

Answer & ExplanationAnswer: Option CExplanation:During +ve part of ViD1 will be forward biasedZener diode is reverse biasedThus net voltage = 6.8 + 0.7 = 4.5 VDuring -ve part of ViD2 will be forward biasedD1 will be reversed biasedThus net voltage = -0.7 V .View Answer Workspace Report Discuss in Forum

33.Four signalsm1 = cos (0t), m2(t) = cos (0t), m3(t) = 2 cos (0t) and m4(t) = cos (4 0t) are multiplexed by Time Division Multiplexing system. The commutator speed in rps is

A.2f0B.3f0

C.4f0D.8f0

Answer & ExplanationAnswer: Option DExplanation:Apply Nyquist criteria.Maximum angular speed is 40Thus commutator speed must be 2 x 42 = 80 .View Answer Workspace Report Discuss in Forum

34.The feedback control system shown in figure is stable for what values of K and T?

A.K < 0, T > - 1

B.K > 0, T > 0

C.K > 0, T > - 1

D.K < 0, T > 0

Answer & ExplanationAnswer: Option CExplanation:Apply Routh Hurwitz criteria.View Answer Workspace Report Discuss in Forum

35.Find z parameters for the following network

A.

B.

C.

D.

Answer & ExplanationAnswer: Option AExplanation:Use Mesh analysisz parameters are given by, V1 = Z11I1 + Z12I2 ...(a)V2 = Z21I1 + Z22I2 ...(b)

Apply KVL to mesh 1V1 = 3I1 - 2I3 ...(i)Apply KVL to mesh 2V2 = 2I2 + 2I3 ...(ii)Apply KVL to mesh 35I3 + 2I2 -2I1 =0 ...(iii)5I3 = 2I1 - 2I2

Put I3 in eq. (i)

...(iv)Put in (ii)Similarly, ...(v)Compare equation (iv) to (a) and equation (v) to (b) .

36.Find the value of ix if each resistance is of 3 and each battery is of 6 V.

A.1.1428 V

B.1.432 A

C.0.2892 A

D.O A

Answer & ExplanationAnswer: Option CExplanation:Using Mesh analysis

Loop 1 : 6 = 9i1 - 3i4 ...(i)Loop 2: 6 + 6 = - 6i2i2 = - 2A ...(ii)Loop 3: 6 = 9i3 - 3i4 ...(iii)Loop 4: 6 = 9i4 - 3i3 - 3i1 ...(iv)Solving the above equations simultaneouslyi1 = i3 = 1.1428 Ai4 = 1.432 A ix = i4 - i3 = 0.2892 A.View Answer Workspace Report Discuss in Forum

37.An image uses 512 x 512 picture elements. Each of the picture elements can take any of the 8 distinguishable intensity levels. The maximum entropy in the above image will be

A.2097152 bits

B.786432 bits

C.648 bits

D.144 bits

Answer & ExplanationAnswer: Option BExplanation:Picture elements (pixel) = 512 x 512 = 262144Information per picture element N = log2 M M = 2N 8 = 2N N = 3 Information for 262144 pixels = 262144 x 3 = 786432 bits Entropy is same as average information per message.View Answer Workspace Report Discuss in Forum

38.Find RTH

A.200

B.0

C.400

D.None of these

Answer & ExplanationAnswer: Option CExplanation:Since there is no independent source in the networkVTH = O VTo find RTh apply : A current source of 1 A across A and B.

Apply KCL at node n1

Apply KCL at node Vn2

Vn1 = 2Vx put in (i)

Vn2 = 4vxput in (ii)

100 + Vx = 2Vx100 = Vx Vn2 = 4Vx = 400 V .View Answer Workspace Report Discuss in Forum

39.If then for this to be true x(t) is __________ .

A.exp (j2pf/t)

B.exp (-j2pf/t)

C.exp (j2pft)

D.exp (-j2pft)

Answer & ExplanationAnswer: Option CExplanation:Consider x(t) = ej2pfty(t) = h(t) * x (t) y(t) = h(t). ej2pf(t - t) . dt= ei2pfth(t) . e- j2pft . dt= ej2pft x H(f) = x(t) x H(f) .View Answer Workspace Report Discuss in Forum

40.The dual transform of the given network as

A.

B.

C.

D.none of the above

Answer & ExplanationAnswer: Option CExplanation:Dual of R G

41.A /4 long high frequency transmission line is terminated into one impedance ZR. If Z0 be the characteristic impedance of the line, then input impedance Zin is

A.ZoZR

B.Z2R/Zo

C.Z20/ZR

D.Infinity

Answer & ExplanationAnswer: Option CExplanation:, here and [Here ZL = ZR].View Answer Workspace Report Discuss in Forum

42.Find the power absorbed or delivered by each source:

A.a = -32 b = - 12 c = -80 d = -60

B.a = -32 b = - 12 c = 80 d = 60

C.a = 32 b = 12 c = -80 d = 60

D.a = -32 b = 12 c = 80 d = -60

Answer & ExplanationAnswer: Option DExplanation:Source a : Power : 8 x (4) = - 32 W. Source A has delivered the power as it is. - ve. Source b: Source B has absorbed power of + 12 W as power is positive Source c: Source C has absorbed power of 5 x 16 = 80 W Source d: Delivers power of 3 x ix x (5) = 3 x 4 x (- 5) = -60 W Total power of the network is - 32 + 12 + 80 - 60 = 0 Watts.View Answer Workspace Report Discuss in Forum

43.The function shown in figure, can represent a probability density function for A __________

A.A = 1/4

B.A = 1/6

C.A = 1/9

D.A = 1/2

Answer & ExplanationAnswer: Option CExplanation:The condition for the probability density function d(x) isand is always non negative or the area under the curve f(x)is unity. => A x 6 +x 2 x 3A = 1 or 9A = 1 or A =.View Answer Workspace Report Discuss in Forum

44.If the transfer function of the following network is

The value of the load resistance RL is

A.R/4B.R/2

C.RD.2R

Answer & ExplanationAnswer: Option CExplanation:

.View Answer Workspace Report Discuss in Forum

45.A bipolar differential amplifier uses a transistor having 0 = 200, ICQ = 100 mA, |Adm| = 500 and CMRR = 80 dB. The value of Rc and RE will be respectively __________ .

A.1.25 k, 1.25 M

B.125 k, 1.25 M

C.1.25 M, 1.25 k

D.3.25 k, 3.25 M

Answer & ExplanationAnswer: Option AExplanation:80 = 20 log CMRR CMRR = 10000

But CMRR = 2gmRE = 1.25 M Adm = - gmRC = 1.25 K.

46.System is at rest for t < 0. Vs = sin 2t. Determine v(t) for t greater than zero.

A.

B.

C.

D.

Answer & ExplanationAnswer: Option DExplanation:KCL at node V

Natural response from s + 2, is Ae-2tFor Vs = sin 2t, s = 2j

Steady state response Since the system is at rest for t < 0

Now substituting t = 0, we get

View Answer Workspace Report Discuss in Forum

47.A 300 MHz plane EM wave is propagating in free space. The wave is incident normally on an infinite copper slap. The antenuation constant for the wave is (copper = 5.8 x 107 mho/ m). The skin depth is

A.0.005 x 10-3 m

B.0.0038 x 10-3m

C.0.0028 x 10-3 m

D.0.0024 x 10-3 m

Answer & ExplanationAnswer: Option BExplanation:.View Answer Workspace Report Discuss in Forum

48.Following figure represents

A.ANDB.OR

C.NANDD.NOT

Answer & ExplanationAnswer: Option CExplanation:

The output C is low only when A and B are both 1.View Answer Workspace Report Discuss in Forum

49.For parallel RLC circuit, which one of the following statements is NOT correct?

A.The bandwidth of the circuit deceases if R is increased

B.The bandwidth of the circuit remains same if L is increased

C.At resonance, input impedance is a real quantity

D.At resonance, the magnitude of input impedance attains its minimum value

Answer & ExplanationAnswer: Option DExplanation:Bandwidth = Thus it is maximum at resonance.View Answer Workspace Report Discuss in Forum

50.The practical resonant circuit is shown in above figure. Find the expression for the resonant frequency o

A.

B.

C.

D.None of these

Answer & ExplanationAnswer: Option BExplanation:

Rationalising,

Equating imaginary parts of numerator to 0, .

SECTION 41.A ramp input applied to a unity feedback system results in 5% steady state error. The type number and zero frequency gain of the system are respectively.

A.1 and 20

B.0 and 20

C.0 and 1/20

D.1 and 1/20

Answer & ExplanationAnswer: Option AExplanation:ess = 5% .05.View Answer Workspace Report Discuss in Forum

2.A fair coil is tossed 10 times. What is the probability that ONLY the first two tosses will yield heads?

A.

B.

C.

D.

Answer & ExplanationAnswer: Option CExplanation:Use ncr, (p)r (q)n-r [for any two losses yield head].

But in present case it is required case it is only for first two tosses. Thus in this case 10 times.View Answer Workspace Report Discuss in Forum

3.The equation sin (z) = 10 has

A.no real or complex solution

B.exactly two distinct complex solutions

C.a unique solution

D.an infinite number of complex solutions

Answer & ExplanationAnswer: Option AExplanation:Since value lies between - 1 and + 1. Therefore no real or complex solution exists.View Answer Workspace Report Discuss in Forum

4.Given If , then the value of K is

A.1B.2

C.3D.4

Answer & ExplanationAnswer: Option DExplanation:

.View Answer Workspace Report Discuss in Forum

5.Consider the Schmitt trigger circuit shown below. A triangular wave which goes from - 12 V to 12 V is applied to the inverting input of the op-amp. Assume that the output of the op-amp moves from +15 V to -15 V. The voltage at the non-inverting switches between

A.-12 V and +12 V

B.-7.5 V and +7.5 V

C.-5 V and +5 V

D.0 V and 5 V

Answer & ExplanationAnswer: Option BExplanation:V0 is switching between + 15 to - 150Thus voltage at non-inverting input switches between, 7.5 to - 7.5 .

6.i1(t) for t > 0 is given by

A.2.5 - 2.5e-2t

B.2.5 - 1.25e-2t

C.1.25 - 1.25e-2t

D.None of these

Answer & ExplanationAnswer: Option BExplanation:The time constant t is given by

As Req = 4 || 4 = 2 i1(t) = iF - (iF - i1)e-t/tifinal = = 2.5 A i1(t) = 2.5 - (2.5 - 1.25)e-t/1/2i1(t) = 2.5 - 1.25e-2t.View Answer Workspace Report Discuss in Forum

7.Find I in 4 resistor.

A.1 A

B.0.5 A

C.0.75 A

D.0.95 A

Answer & ExplanationAnswer: Option CExplanation:Apply KVL to first to first loop6i1 - 2i2 = 10(4 + R)i2 - 2i1 - 2i3 = 04i3 - 2i2 = 0but i3 = 0.5 A2 - 2i2 = 0 i2 = 1 A, 6i1 - 2 = 10 i1 = 2 A, (4 + R) - 2 X 2 -1 = 0 R = 1 Using Nodal analysis for loop 2At node A, VA - 20 + VA + VA - VB = 03VA - VB - 20 = 03VA - VB = 20 ...(i)At node B, 2VB - 2VA + 2VB + VB = 05VB - 2VA = 0 ...(ii)Multiplying (i) by 2 and (ii) by 36VA - 2VB = 40- 6VA - 15VB = 013VB = 40VB 3 V View Answer Workspace Report Discuss in Forum

8.Which of the following Boolean Expression correctly represents the relation between P, Q, R and M1?

A.M1 = (P OR Q) XOR R

B.M1 = (P AND Q) XOR R

C.M1 = (P NOR Q) XOR R

D.M1 = (P XOR Q) XOR R

Answer & ExplanationAnswer: Option DExplanation:M1 = Z XOR RM1 = (X, Y) XOR R= M1 = [P.Q.(P + Q)]XOR R= (PQ + PQ) XOR R = (P XOR Q) XOR R.View Answer Workspace Report Discuss in Forum

9.A sequential multiplexer is connected as shown in figure. Each time the multiplexer receives the clock, it switches to the next channel from 6 to 1. If the input signals are :A = 10 cos 2p(4 x 103t)B = 15 cos 2p(5 x 103t)C = 20 cos 2p(6 x 103t)D = 10 cos 2p(10 x 103t)The minimum clock frequency should be __________ kHz.

A.8 kHz

B.50 kHz

C.20 kHz

D.10 kHz

Answer & ExplanationAnswer: Option CExplanation:Minimum clock freq. = 2fmWhere fm is highest freq. component = 2 x 10 kHz = 10 kHz.View Answer Workspace Report Discuss in Forum

10.The Boolean function realized by the logic circuit shown is

A.F = m(0, 1, 3, 5, 9, 10, 14)

B.F = m(2, 3, 5, 7, 8, 12, 13)

C.F = m(1, 2, 4, 5, 11, 14, 15)

D.F = m(2, 3, 5, 7, 8, 9, 12)

Answer & ExplanationAnswer: Option DExplanation:

F = m (2, 3, 5, 7, 8, 9, 15).

11.Find RAB

A.1

B.

C.

D.2

Answer & ExplanationAnswer: Option DExplanation:By solving

View Answer Workspace Report Discuss in Forum

12.For the network shown below, determine fHi and fHO, if, r0 = , Cbe = 36 Pf, Cbc = 4 Pf, Cce = 1 Pf, Cwo = 8 Pf, Cwi = 6 Pf

where Cwo = wiring capacitance at output side, Cwi = wiring capacitance at input side

A.738.24 KHz, 8.6 MHz

B.781.2 KHz, 10.6 MHz

C.838.32 KHz, 9.2 MHz

D.938.24 KHz, 8.2 MHz

Answer & ExplanationAnswer: Option AExplanation:Equivalent is

RTi = RS || R1 || R2 || Ri = 0.531 K Ci = Cwi + Cbe + (1 + AV)Cbc= 6pf + 36pf + (1 - c - 90) 4 pf = 460 pf

RT2 = RC || RL = 4 K || 2.2 K = 1.419 KW C0 = Cwo + Cce + CMO

View Answer Workspace Report Discuss in Forum

13.Complex pole in z-plane is as shown below. This is the pole diagram for __________ .

A.Decaying sinusoidal sequence

B.Growing sinusoidal sequence

C.Decaying non sinusoidal sequence

D.Growing non sinusoidal sequence

Answer & ExplanationAnswer: Option AExplanation:Decaying sinusoidal sequence.View Answer Workspace Report Discuss in Forum

14.Four messages band limited to W, W, 2W and 3W respectively are to be multiplex using Time Division Multiplexing (TDM). The minimum bandwidth required for transmission of this TDM signal is

A.WB.3W

C.6WD.7W

Answer & ExplanationAnswer: Option CExplanation:In TDM minimum bandwidth is twice the maximum frequency present. Thus, 6W is the answer.View Answer Workspace Report Discuss in Forum

15.Which of the following terms are equivalent? 1. A B C2. A B C3. A B C4. A B C

A.1 and 4; 2 and 3

B.1 and 3

C.2 and 3

D.1, 2 and 4

Answer & ExplanationAnswer: Option AExplanation:A B C = A (B C) + A(B C) = A(B C) + A (B C) = A B C A B C = A (B C) + A(B C) = A(B C) + A(B C) = A B C.

16.If 137 + 276 = 435 how much is 731 + 672?

A.534B.1403

C.1623D.1513

Answer & ExplanationAnswer: Option CExplanation:Given, 137 + 276 = 435Adding units digits i.e., 7 + 6 = 13, but given as 5, which is 13 - 8 and also 1 is carry forwarded to the tens digit.i.e., + 1

Here, 7 + 3 + 1 = 1 i.e., 11 - 8 = 3 and 1 is carry forwarded to hundred digits

Now, the sum of digits in hundred's place is 1 + 1 + 2 = 4ie.,

Using the same logic, we have

Using digits sum 1 + 2 = 3 Tens digits sum = 3 + 7 = 10 i.e., 10 - 2 and + 1 carry forward Hundreds digits sum = 1 + 7 + 6 = 14 i.e., 14 - 8 = 6 and one carry forward.View Answer Workspace Report Discuss in Forum

17.An astable multivibrator circuit using IC 555 timer is shown below. Assume that the circuit is oscillating steadily

The voltage Vc across capacitor varies between

A.3 V to 5 V

B.3 V to 6 V

C.3.6 V to 6 V

D.3.6 V to 5 V

Answer & ExplanationAnswer: Option BExplanation:Charging and discharging level of capacitor will be the voltage across it. This is equal to Vcc and Vcc. Thus 3V to 6V is the voltage VC across the capacitor.View Answer Workspace Report Discuss in Forum

18.The fourier transform of a double sided exponential functionis

A.

B.

C.

D.

Answer & ExplanationAnswer: Option DExplanation:Consider general expressionX(j) = ea|t| e- jt dt= eat e-jt dt + e-ate-jt dtX(j) = e(a - j)t dt + e -(a + j)t dt

Since .View Answer Workspace Report Discuss in Forum

19.The number of product terms in the minimized sum of product expression obtained through the following k map is (where "d" denotes don't care states)

A.2B.3

C.4D.5

Answer & ExplanationAnswer: Option BExplanation:

Y = A B D + A B D + A C D.View Answer Workspace Report Discuss in Forum

20.If A = then |A50| will be

A.(1 - 1002)

B.(1 - 502)

C.(1 - 1002)

D.(1 - 502)

Answer & ExplanationAnswer: Option BExplanation:An = ? Every n x n matrix satisfy its characteristic equation |A - I| = 0 -> eigen vector A - I = |A - I| = = 0 1 =, f(A) = An = 0I + 1A Replace A by 1, I by 1 f() = n = 0 + 1 Differentiate w.r.t. nn - 1 = 1 1 = 0 = n - x 1 = 0 = - = [1 - n] An = [1 - n]+ . 2n An = = A50 = |A50| = (1 - 502).

21.A uniform plane magnetic wave incident normally on a plane surface of a dielectric material is reflected and the percentage of reflected power is 75% What is VSWR?

A.14B.1.4

C.15D.13

Answer & ExplanationAnswer: Option AExplanation:

View Answer Workspace Report Discuss in Forum

22.Silicon is doped with boron to a concentration of 4 x 1017 atoms/cm3. Assuming the intrinsic carrier concentration of silicon to be 1.5 x 1010/cm3 and the value of to be 25mV at 300K. Compared to undoped silicon, the Fermi level of doped silicon

A.goes down by 0.13 eV

B.goes up by 0.13 eV

C.goes down by 0.427 eV

D.goes up by 0.427 eV

Answer & ExplanationAnswer: Option CExplanation:Use EF - Ev = Since it is doped with acceptor impurity, Fermi level will shift down.View Answer Workspace Report Discuss in Forum

23.The Fourier series of a real periodic function has only 1. Cosine terms if it is even2. Sine terms if it is even3. Cosine terms if it is odd4. Sine terms if it is oddWhich of the above statements are correct?

A.(1) and (4)

B.(1) and (2)

C.(2) and (4)

D.(2) and (3)

Answer & ExplanationAnswer: Option AExplanation:, then it has 0 and HZ frequency component.View Answer Workspace Report Discuss in Forum

24.A communication channel with AWGN operating at a signal to noise ratio SNR >> 1 and bandwidth B has capacity C1. If the SNR is doubled keeping B constant, the resulting capacity C2 is given by

A.C2 2C1

B.C2 C1 + B

C.C2 C1 + 2B

D.C2 C1 + 0.3B

Answer & ExplanationAnswer: Option BExplanation:C = B log2[1 + SNR]C = B log2[SNR]C' = B log2[2SNR] = B log2 SNR + B log22C' = C1 + B.View Answer Workspace Report Discuss in Forum

25.Power gain of antenna equals directive gain in VHF and UHF range if and only if efficiency of the antenna is __________ .

A.0.25B.0.5

C.0.75D.1

Answer & ExplanationAnswer: Option DExplanation:Directive gain and power gain are identical except that power gain takes into account the antenna losses. It may be written asGp = Gd => efficiencyIf = 1 , GP = Gd .

26.For the asymptotic Bode magnitude plot shown below, the system transfer function can be

A.

B.

C.

D.

Answer & ExplanationAnswer: Option AExplanation:.View Answer Workspace Report Discuss in Forum

27.A square in which each (i, j)th element of matrix is equal to the negative of the conjugate complex of (j, i)th element, is called

A.Hermitian matrix

B.Skew symmetric matrix

C.Conjugate anti-symmetric matrix

D.Skew-Hermitian matrix

Answer & ExplanationAnswer: Option DExplanation:When aij = - . This matrix is called skew hermitian matrix.View Answer Workspace Report Discuss in Forum

28.Mid-band gain =

A.100

B.- 100

C.50

D.- 50

Answer & ExplanationAnswer: Option BExplanation:

.View Answer Workspace Report Discuss in Forum

29.The characteristic equation of a feedback control system is given by s3 + 5s2 + (K + 6)s + K = 0where K > 0 is a scalar variable parameter. In the root-loci diagram of the system the asymptotes of the root-locus for large values of K meet at a point in the s-plane, whose coordinates are :

A.(- 3, 0)

B.(- 2, 0)

C.(- 1, 0)

D.(2, 0)

Answer & ExplanationAnswer: Option CExplanation:Apply R-H, and find by auxiliary equation.View Answer Workspace Report Discuss in Forum

30.The input 3-dB frequency is :

A.100 Hz

B.148 Hz

C.100 kHz

D.None

Answer & ExplanationAnswer: Option BExplanation:Input 3 dB frequency C = 0.40 F.

31.Consider a stable and causal system with impulse response h[n] and rational system function H(z). Suppose it is known that H(z) contains a pole at z = 1/2 and a zero some where on the unit circle. The precise number and locations of all of the other poles is unknown.The following statement which is false is __________ .

A.H(ej) = 0 for some

B.

C.g[n] = {n[h[n * h[n]} is the impulse response of a stable system

D.h[n] has a finite duration

Answer & ExplanationAnswer: Option DExplanation:Statement (d) is false because a finite-duration sequence must have an ROC that includes the entire z-plane, except possible z = 0 and/or z = This is not consistent with having a pole at z = 1/2.Statement (b) is true corresponds to the value of the z-transform of h[n] at z = 2. Thus, its convergence is equivalent to the point z = 2 being in the ROC. Since the system is stable and causal, all of the poles of H(z) are inside the unit circle, and the ROC includes all the points outside the unit circle, including z = 2.Statement (c) is true. Since the system is causal, h[n] = 0 for n < 0. Consequently, h[n] * h[n] = 0 for n < 0; i.e., system with h[n] * h[n] as its impulse response is causal. The same is then true for g[n] = n[h[n]]. Furthermore, by the convolution property the system function corresponding to the impulse response h[n] * h[n] is H2(z), and by the differentiation property the system function corresponding to g[n] is

From above equation we can conclude that the poles of G(z) are at the same locations as those of H(z), with the possible exception of the origin. Therefore, since H(z) has all its poles inside the unit circle, so must G(z). It follows that g[n] is the impulse response of a causal and stable system.Statement (a) is true because there is a zero on the unit circle.View Answer Workspace Report Discuss in Forum

32.When transient and steady state response require improvement; a __________ is used.

A.Lead compensator

B.Lag - Lead compensator

C.Lag compensator

D.Integrator

Answer & ExplanationAnswer: Option BExplanation:When improvement in transient state response is required Lead compensator is used. When improvement in steady state response is required lag compensator is used. Use of lag-lead compensator increases the low frequency gain which improves the steady state while at the same time it increases bandwidth of the system making the system response very fast.View Answer Workspace Report Discuss in Forum

33.For the collector-feedback amplifier shown below the voltage gain Av will be __________ .

A.-113.07B.-103.07

C.-107.07D.-93

Answer & ExplanationAnswer: Option BExplanation:

= - 103.07 RF -> feedback resistor.View Answer Workspace Report Discuss in Forum

34.Narrow band FM signal can be represented as

A.A cos (2pfct) - A sin (2pfct) sin (2pfmt)

B.A cos (2pfmt) - A sin (2pfct) sin (2pfmt)

C.A cos (2pfct) + A sin (2pfct) sin (2pfmt)

D.A cos (2pfmt) + A sin (2pfmt) sin (2pfct)

Answer & ExplanationAnswer: Option AExplanation:General expression for narrowband FM is A cos (2pfct) - A sin (2pfct) sin (2pfmt).View Answer Workspace Report Discuss in Forum

35.2's complement representation of a 16 bit number (one sign bit and 15 magnitude bits) is FFFF. Its magnitude in decimal representation is

A.0

B.1

C.32, 767

D.65, 535

Answer & ExplanationAnswer: Option BExplanation:First bit is sign bit, hence its magnitude its 2s complement Sign bit is part of magnitude, so it may be 1.

36.In the following transistor circuit VBE = 0. 7 V, rc = 25mV/IE, and and the capacitance are very large

The value of DC current IE is

A.1 mA

B.2 mA

C.5 mA

D.10 mA

Answer & ExplanationAnswer: Option AExplanation:Design is independent of IB as is very high IB 0

Thus applying Kirchhoff's law is base emitter, we get.View Answer Workspace Report Discuss in Forum

37.A certain JK FF has tpd = 12 nsec. The largest MOD counter that can be constructed from these FFs and still operate up to 10 MHz is

A.anyB.8

C.256D.10

Answer & ExplanationAnswer: Option BExplanation:

f0 = 107 Hz 8.33n = 8.View Answer Workspace Report Discuss in Forum

38.A second-order system has a transfer function is given by . If the system, initially at rest is subjected to a unit step input at t = 0, the second peak in response will occur at :

A.p sec

B.p/3 sec

C.p/6 sec

D.p/2 sec

Answer & ExplanationAnswer: Option CExplanation:

Peak time

.View Answer Workspace Report Discuss in Forum

39.Value of ZL for maximum power transfer is

A.R

B.R + jL

C.R - jL

D.None of these

Answer & ExplanationAnswer: Option CExplanation:The value of load for maximum power transfer is given by complex conjugate of ZAB. ZAB = R + jXL = R + jL. ZL for maximum power transfer is given by, ZL = R - jL.View Answer Workspace Report Discuss in Forum

40.A periodic voltage has following values for equal time intervals changing suddenly from one value to next : 0, 5, 10, 20, 50, 60, 50, 20, 10, 5, 0, - 5, - 10 etc.RMS value of the waveform is :

A.31 v

B.32 v

C.insufficient data

D.none of these

Answer & ExplanationAnswer: Option AExplanation:Mean value of V2 == 965 V rms value = 31 V.

41.The magnitude plot of a rational transfer function G(s) with real coefficients is shown below. Which of the following compensators has such a magnitude plot?

A.Lead compensator

B.Lag compensator

C.PID compensator

D.Lead-lag compensator

Answer & ExplanationAnswer: Option DExplanation:Since magnitude plot shows both increasing as well as decreasing plot, it is lead-lag compensator.View Answer Workspace Report Discuss in Forum

42.The circuit shown in the figure converts

A.BCD to Binary code

B.Binary to excess - 3 code

C.Excess - 3 to Gray

D.Gray to Binary

Answer & ExplanationAnswer: Option DExplanation:Let input is 1 1 0 1y1 = x1

y2 = x2 x1 1 1 = 0y3 = x3 x2 = 0 0 = 0y4 = x4 x3 = 1 0 = 1if input is gray then y1 y2 y3 y4 will be Binary.View Answer Workspace Report Discuss in Forum

43.Using 4-bit numbers (n = 4) if k = (0011)2 how is k expressed in 2's complement.

A.(1011)2B.(1101)2

C.(1100)2D.(0101)2

Answer & ExplanationAnswer: Option BExplanation:In 2's complementSimply find 2s complement (0011)2 1100 + 1 (1101)2 .View Answer Workspace Report Discuss in Forum

44.Which of the following options is the closest in meaning to the world below:Circuitous

A.cyclicB.indirect

C.confusingD.crooked

Answer & ExplanationAnswer: Option BExplanation:Circuitous means round about or not direct. Therefore the closest in meaning will be indirect.View Answer Workspace Report Discuss in Forum

45.System is :

A.causal

B.stable

C.non causal

D.anticausal

Answer & ExplanationAnswer: Option AExplanation:ROC is towards infinity hence the signal is causal. Now the ROC does not contain the unit circle hence the system is unstable.

46.A sphere with charge Q was placed in, but not touching a larger hollow sphere. If there is any charge initially on the outer sphere; to remove it the outer sphere was earthed momentarily, and then the inner sphere was removed. The charge remaining on the outer sphere was then measured. This charge was found to be __________ to the charge on the inner sphere.

A.equal and of same again

B.equal and of opposite sign

C.unequal and of same sign

D.unequal and of opposite sign

Answer & ExplanationAnswer: Option BExplanation:This is true for all sizes of the spheres and for all types of dielectric media between the spheres.View Answer Workspace Report Discuss in Forum

47.At t = ; VR2 and VC will be

A.4, 4

B.2, 4

C.0, 2

D.2, 0

Answer & ExplanationAnswer: Option BExplanation:When steady state is reached again; then C is openNow VR2 = 2V.View Answer Workspace Report Discuss in Forum

48.Two products are sold from a vending machine, which has two push buttons P1 and P2. When a button is pressed, the price of the corresponding product is displayed in a 7-segment display. If no buttons are pressed, '0' is displayed, signifying 'Rs' .0'. If only P1 is pressed, '2' is displayed, signifying 'Rs. 2'.If only P2 is pressed, '5' is displayed, signifying 'Rs. 5'.If both P1 and P2 are pressed, 'E' is displayed, signifying 'Error'. The names of the segments in the 7-segment display, and the glow of the display for '0', '2', '5' and 'E', are shown below.

Consider I. push button pressed/not pressed is equivalent to logic 1/ 0 respectivelyII. a segment a to g are considered as functions of P1 and P2, then which of the respectively.If segments a to g are considered as functions of P1 and P2, then which of the following is correct?

A.g = P1 + P2, d = c + e

B.g = P1 + P2, d = c + e

C.g = P1 + P2, e = b + c

D.g = P1 + P2, e = b + c

Answer & ExplanationAnswer: Option BExplanation:From the figure

View Answer Workspace Report Discuss in Forum

49.Laplace transform of {t cos at} is:

A.

B.

C.

D.

Answer & ExplanationAnswer: Option AExplanation:x(t) x(s) .View Answer Workspace Report Discuss in Forum

50.The following series RLC circuit with zero initial conditions is excited by a unit impulse function (t).

For t > 0, the output voltage Vc (t) is

A.

B.

C.

D.

Answer & ExplanationAnswer: Option DExplanation:

.

SECTION 51.Determine the potential difference between the points A and B in the steady state

A.40 V

B.60 V

C.25 V

D.75 V

Answer & ExplanationAnswer: Option DExplanation:

View Answer Workspace Report Discuss in Forum

2.Find the value of K and velocity constant Kv so that the maximum overshoot in the unit step response is 0.2 and the peak time is 1 sec.

A.12.37, 0.175

B.12.86, 2.175

C.11.36, 1.175

D.1.86, 0.175

Answer & ExplanationAnswer: Option AExplanation:

2 = 0.259(1 - 2) = 0.259 - 0.25921.2592 = 0.2592 = 0.206 = 0.45

2n = K n = K and 2n = 1 + KKv

n = KK = 2nK = 12.371 + 12.37Kv = 2 x 0.45 x 3.52Kv = 0.175 .View Answer Workspace Report Discuss in Forum

3.The transfer function of a discrete time LTI system is given by

Consider the following statements:S1: The system is stable and causal for ROC: |z| > 1/2S2: The system is stable but not causal for ROC: |z| < 1/4S3: The system is neither stable nor causal for ROC: 1/4 < |z| < 1/2Which one of the following statements is valid?

A.Both S1 and S2 are true

B.Both S2 and S3 are true

C.Both S1 and S3 are true

D.S1, S2 and S3 are all true

Answer & ExplanationAnswer: Option CExplanation:S1 true, S2 False (meter stable nor causal) S3 True.View Answer Workspace Report Discuss in Forum

4.In the following network, the switch is closed at t = 0- and the sampling starts from t = 0. The sampling frequency is 10Hz.

The expression and the region of convergence of the z-transform of the sampled signal are

A.|z| < e-5

B.|z| < e-0.05

C.|z| < e-0.05

D.|z| > e-5

Answer & ExplanationAnswer: Option CExplanation:X(Z) = 5.e-0.05n.Z-n

.View Answer Workspace Report Discuss in Forum

5.The logic function implemented by the following circuit at the terminal OUT is

A.P NOR Q

B.P NAND Q

C.P OR Q

D.P AND Q

Answer & ExplanationAnswer: Option DExplanation:AND gate.

6.In a transmission line terminated with a load equal to the characteristic impedance, the reflection coefficient is

A.0B.+1

C.-1D.Infinity

Answer & ExplanationAnswer: Option AExplanation:.View Answer Workspace Report Discuss in Forum

7.Consider a control system shown in given figure. For slight variation in G, the ratio of open loop sensitivity to closed loop sensitivity will be given by

A.1 : (1 + GH)

B.1 : (1 + GH)- 1

C.1 : (1 - GH)

D.1 : (1 - GH)- 1

Answer & ExplanationAnswer: Option BExplanation:Open loop sensitivity = 1 Closed loop sensitivity = Hence ratio = 1 : (1 + GH) - 1.View Answer Workspace Report Discuss in Forum

8.Consider the signal S(t) shown below :

The slope of the matched filter output during the interval

A.8B.12

C.16D.18

Answer & ExplanationAnswer: Option AExplanation:Match filter will be

The output of this will be convolution of S(t) with match filter.

.View Answer Workspace Report Discuss in Forum

9.The logic function f(A B, C) = m(0, 2, 4, 5, 6) represented by

A.i

B.i, ii

C.i, iii

D.i, ii, iii

Answer & ExplanationAnswer: Option BExplanation:

View Answer Workspace Report Discuss in Forum

10.An electrical system and its signal-flow graph representation are shown in figure (a) and (b) respectively

The values of G2 and H, respectively are

A.

B.

C.

D.

Answer & ExplanationAnswer: Option AExplanation:Vi(s) = Z1I1 + Z3(I1 - I2) ...(i)V0(s) = Z4I2I2(Z3 + Z2 + Z4) = Z3I1 ...(ii)From signal flow graphI2 = I1G2 Hence, ...(iii)From SFG ...(iv)From (ii), (iii), (iv) we get H.

11.Consider the circuit shown below of 2 : 1 MUX is given by the function g = ac + bc

Then f is

A.X X

B.XY + E X Y

C.XE + XY

D.None of these

Answer & ExplanationAnswer: Option BExplanation:g = ac + bcmin c = 0, g = b and c = 1, g = aSo f = y + ya y x + yg ...(i)g = x b + x.a = x E + X.o x Efrom (i) f = y x + y x E.View Answer Workspace Report Discuss in Forum

12.For the given phase lead network, the maximum possible phase lead is

A.sin-1

B.30

C.45

D.60

Answer & ExplanationAnswer: Option BExplanation:The maximum phase lead is given by, sin m = Where Hence sin m = Hence m = 30.View Answer Workspace Report Discuss in Forum

13.Given (224)r = The value of the radix 'r' is :

A.5B.6

C.7D.10

Answer & ExplanationAnswer: Option AExplanation:(224)r = = (13)r x (13)r 2r2 + 2r + 4 = (r + 3) (r + 3) = r2 + 6r + 9 r2 - 4r - 5 = 0 (r - 5) (r + 1) = 0 r = - 1 or r = 5 Negative base is not possible. Therefore base of the equation is 5.View Answer Workspace Report Discuss in Forum

14.Which factor determines whether the medium is free space, lossless dielectric, lossy dielectric or good conductor?

A.Attenuation constant

B.Constitutive parameters (, , )

C.Loss tangent

D.Reflection coefficient

Answer & ExplanationAnswer: Option CExplanation:Loss tangent (i) for lossless dielectric, tan 1(ii) for good conductor, tan >> 1 (iii) for lossy dielectric, tan = 1.View Answer Workspace Report Discuss in Forum

15.Find RAB

A.2

B.1.5

C.3

D.None of the above

Answer & ExplanationAnswer: Option AExplanation:Reduced Diagram :

RAB = 3 || 6 = 2

16.In the following circuit, the switch S is closed at t = 0 The rate of change of current (0 +) is given by

A.0

B.

C.